Saturday, January 01, 2011

God, Science, and Metal Detectors

A redated post.

This is from my response to the Carrier-Wanchick debate.

Carrier relied on an argument from displacement, but it is the nature of science not to look for supernatural causes, or if so, to accept them as a last resort. It is like saying that the $100 bill you lost at the beach must have been stolen because after scouring the area with a metal detector, you didn't find it. Science is extremely good at telling us some things we need to know; there are other things it is not so good at, and it is far from proven that we ought to make science the measure of all things.



Jan 1, 2011 addendum: Methodological naturalism in the sciences actually weakens the argument for atheism based on science. The only way scientific evidence could possibly undermine religious belief would be if scientific evidence could have gone the other way and confirmed religious belief.

103 comments:

Robert said...

Hello Victor,

“Science is extremely good at telling us some things we need to know; there are other things it is not so good at, and it is far from proven that we ought to make science the measure of all things.”

I always saw “scientism”, the claim that science is the “measure of all things”, that science alone leads to truth and reveals truth, as being a laughable example of a self –refuting statement.

I mean how do you prove SCIENTIFICALLY that science ought to be the measure of all things?

You can’t use “science” to prove science should have this function.

I mean what scientific experiment will you perform to confirm or disconfirm this claim?

Caveat – I love science and highly respect scientists. That being said however, science only deals with a part of the pie that we call reality. If it goes beyond its proper realm it oversteps its boundaries. I don’t understand why more people do not take Dirty Harry more seriously when he tells us:

“A man’s got to know his limitations!”

Robert

Blue Devil Knight said...

I like the metal detector metaphor, it is better than the "drunk looking for his keys under the lamp" metaphor.

Blue Devil Knight said...

The only way scientific evidence could possibly undermine religious belief would be if scientific evidence could have gone the other way and confirmed religious belief.

Natural philosophers used to mix in theological explanations all the time. If you look at the history of the study of life, it certainly could have gone the other way. Indeed it did for centuries. This cycle happened countless times (e.g., with astronomy, geology, weather, etc)..

MN is not a dogma, but a well-earned contingent fact. People who say it is a priori don't know the history of science. Sure, it now may SEEM a priori, but it didn't always!

Anonymous said...

MN is not a dogma, but a well-earned contingent fact. People who say it is a priori don't know the history of science. Sure, it now may SEEM a priori, but it didn't always!

It's almost as if each and every time something popped up that didn't fit with naturalism (action at a distance, quantum physics, the big bang, etc.) it ended up being called "natural" later on in spite of it all.

MN is a misnomer, because naturalism itself doesn't have much of a definition other than "not God", or more particularly, "not the christian God".

Blue Devil Knight said...

Anon: even if it were such a negatively defined dictum, it's nontrivial. "No gods intervening" would have been enough to get you burned at the stake until relatively recently in our history.

Also, it isn't just that negatively defined dictum. It also includes other methods, such as a lack of respect for authority, especially revelation without further evidence. Among other things (e.g., the impulse to test your theories with experiments, to express them mathematically, etc).

Anonymous said...

even if it were such a negatively defined dictum, it's nontrivial. "No gods intervening" would have been enough to get you burned at the stake until relatively recently in our history.

And "gods intervening" would have gotten you killed or thrown into a re-education camp potentially even more recently in history. Further, even those times where saying "no gods intervening" would have resulted in a penalty, the injunction wasn't some broad universal writ against scientific or philosophical speculation, or else Aquinas would have been burned at the stake for suggesting God works through secondary causes.

Also, it isn't just that negatively defined dictum. It also includes other methods, such as a lack of respect for authority,

"Methodological naturalism" entails a lack of respect for authority? L O L

Oh wait, you meant naturalism does? Let me laugh even harder!

especially revelation without further evidence. Among other things (e.g., the impulse to test your theories with experiments, to express them mathematically, etc).

And testing theories with experiments is contrary to non-naturalistic ideas how exactly? Methodological naturalism is a flub, because "naturalism" means whatever someone wants it to mean at the time. About the one thing forbidden is "no reference to God, specifically the Christian God, at all", which backs up Victor's point. I'm going a step further and pointing out the idea is shallow even on other levels.

Thrasymachus said...

I'm also not sure methodological naturalism would weaken a case for science against God (not that I think there is much of one).

For MethNat gives credence for MetaNat. If naturalism is true, then it is certain there is a complete science available with methodological naturalism; if Theism is true, it is not certain (for my money, unlikely) that there is a complete science available with methodological naturalism. So if we're confident that we'll get a complete science with MethNat, that is evidence in favour of MetaNat.

Victor Reppert said...

I think the idea of a complete science needs clarification here, but I assume that completeness means getting all the causes.

But someone could certainly accept MN as a matter of convention, without necessarily thinking that we can every hope to get all the causes through MN. We could just adopt it as the right rule for a particular stage of inquiry, defeasible of course, which might be set aside when the situation changes. We might say something like this: non-natural causes are probably going to be harder for science to get a grip on than natural causes, so let's map out all the natural causes first and see what's left over after we're done.

Anonymous said...

Thrasymachus said:

"If naturalism is true, then it is certain there is a complete science available with methodological naturalism"

Why? There are many kind of "naturalisms". For example, if naturalism is true, then our world could be for example (1) full of gaps (and without causality) or (2) gapless, and everything between these two alternative. In the first case complete science maybe wouldn't be available with methodological naturalism. So naturalism doesn't imply that "it is certain there is a complete science available with methodological naturalism"

Anonymous said...

Vic, this analogy assumes we know there was a $100 bill that was lost in the first place. If we knew that to be the case scientists would never used metal detectors to find it. But let's grant this anyway and use metal detectors as a metaphor for science, which is what you're doing. Then what alternative method do you propose for finding that $100 bill? Telepathy? Clairvoyance? A strong feeling?

William James proposed that we should meet the god-hypothesis halfway, but his method will justify any god-hypothesis of the investigator.

Science detects things by empirical means which includes the five senses. These are the only senses we know.

Science focuses on that which is detectable and its method is doubt. It is the skeptical adult approach rather than the gullible childish approach to an issue that if a god existed he would be pleased with. If God is not detectable and if he cannot be apprehended by such an adult attitude then I can't help him.

Victor Reppert said...

Metaphysical reasoning. Reasoning that is appropriate to subjects where a direct experimental solution is not available.

Is attempting to discover the transcendental presuppositions of science scientific reasoning. If we ask the question "If people actually do science, what must be true of them?" can we settle that question by the methods of some particular science or other?

Anonymous said...

One word: Pragmatism.

Science works and because it does it more than justifies itself.

No other method works otherwise there would be more agreement than there is among metaphysicians.

Blue Devil Knight said...

Anon "no gods" is nontrivial. That was my point, and it is trivially true.

In addition to that edict, the other methodological maxims that scientists tend to follow (e.g., genuflection to evidence over theory, striving for quantification, attempts to falsify theories) are also contingent, but part of the loose cluster of properties that characterizes MN.

There is no single binary criterion that defines MN or science. This doesn't imply that science is the same as non-science, just that the category boundary can sometimes be unclear. Just like baldness, that doesn't mean it is fictitious.

Victor Reppert said...

Pragmatism doesn't answer my question. I asked what do human being have to be like in order for science to be even possible?

Science often works because it waves aside certain issues and problems. The problems don't go away, however, when you wave them aside.

What science typically ignores is the fact that while there is an object of science that the scientist analyzes, there is also the subject that does the science. Why are we the sorts of creatures that can do science?

We might ask, as Wigner did, "Why do we live in a universe in which mathematics allows us to know the truth about nature?" How do we explain what he called the unreasonable effectiveness of mathematics? Is this a scientific question?

And isn't it our acculturation that prevents all of us from saying "Look, basically science sucks. It's a massively misleading way of looking at the world. Sure, it gets some practical results, but reality could be different from what science reveals. For example, if we were brains in vats, the vatkeepers could cause us to engage in "science" to connect up the appearances the vatkeepers are feeding us. But that doesn't mean that we are getting the truth, only that we are perceiving regularities in the "reality" the keepers are pumping into our brains."

Typically, those who make science the measure of all things have to punt to future science in order to have any assurance that the answers they are looking for are forthcoming. But predicting the future trajectory of science is tricky business. Who would have predicted that Newtonian theory would have problems? Who would have predicted quantum theory, or big bang cosmology? The idea that the history of science promises us more and more successful reductions, as opposed to reductive failures (which can be a kind of scientific progress as well), strikes me as naive.

Pragmatism is a typically American brand of philosophy, I might add. The fact that you are inclined toward pragmatism is the result of your indoctrination as an American. If you were born in India, I can guarantee that you wouldn't be a pragmatist. Maybe you should take the outsider test for epistemology. (OTE).

Blue Devil Knight said...

I am a methodological pluralist but ontological naturalist. Literature is still ahead of psychology and neuroscience in revealing certain aspects of the human condition. I'd throw in cinema, music, other works of art too.

I don't need science to tell me if I love my daughter, even though that love is a natural phenomenon.

Qua scientist though, I am a methodological naturalist. I can't publish a paper justifying a result with 'It's just obvious, I don't need science to prove this!' (Though that would make for a funny line in a paper).
Science detects things by empirical means which includes the five senses. These are the only senses we know.

Well, six senses really, as vestibular sense is not reducible to the traditional five.

Blue Devil Knight said...

Not many papers use data from the vestibular system, though. :)

Old school chemistry was awesome, they would report the taste of compounds. They still often mention the smell.

Blue Devil Knight said...

As Penelope Maddy said, wisely, of the scientist studying science:
"[S]uppose she is engaged in her scientific study of science: she calls on her physiological, psychological, neurological accounts of human perception and conceptualization, her linguistic, psycholinguistic, cognitive scientific theories of the workings of human language, her physical, chemical, astronomical, biological, botanical, geological descriptions of the world in which these humans live; she uses these and any other relevant scientific findings to explain how these humans, by these means, come to know about this world."

What else are you to do in your study of science? Use sub-par methods and knowledge of the world, gaind from some anachronistic point of view that thinks science needs grounding by an independent court of epistemic appeal? Sorry, Mr Bayesian, you should know better than that. This is a post-Quinean world you are living in.

Blue Devil Knight said...

Victor, can you point to a single unqualified success story that has come from the application of this supposedly powerful transcendental method you are so gung-ho about?

Perhaps Kant's argument about Euclidean space? :)

Anonymous said...

Anon "no gods" is nontrivial. That was my point, and it is trivially true.

And yet, Aquinas and others who stressed the reality of secondary causes managed to survive. Miracle, I suppose.

In addition to that edict, the other methodological maxims that scientists tend to follow (e.g., genuflection to evidence over theory, striving for quantification, attempts to falsify theories) are also contingent, but part of the loose cluster of properties that characterizes MN.

No, they're part of the loose cluster of properties that characterizes science, and which gets the label of '(methodological) naturalism' slapped onto it by naturalists and some of the slower theists.

Again, what forbids a non-naturalist from favoring quantification in a particular investigation? Or falsification? There is nothing distinctly "naturalist" about these methods. (As for 'evidence over theory', please. That one's violated constantly.)

There is no single binary criterion that defines MN or science.

There's not even a criterion that reliably defines naturalism aside from what I've said. It's been redefined over and over, with each new discovery that didn't fit in with the old definition of naturalism being included after the fact. At this point, naturalism means little more than 'Not God, certainly not the Christian God'.

Suggesting that methodological naturalism is the standard for science, while at the same time having it clear that "naturalism" is absolute word-putty, makes the whole claim a joke. MN simply doesn't have much to it, nor does naturalism. Sad, but true.

Blue Devil Knight said...

Anon: Aquinas was a deist, eh? Claiming that some causes are secondary is not the same as saying that all causes are secondary.

My point, trivially true, is that the claim that gods don't intervene in nature is not some empty trivial claim. Supernatural theories of life, speciation, geology, astronomy, mental illness, consciousness, morality, reason etc etc would beg to differ. But perhaps you think such theories were never worth taking seriously.

If that is "not having much to it" then fine, we disagree.

I have listed multiple criteria aside from 'no gods' that characterize a more scientific approach. You haven't addressed them. I recommend reading a few papers from a scientific journal, and compare them to articles from a philosophy/religious studies journal. The differences are stark and undeniable, even though there may be fuzzy cases.

The existence of hermaphrodites doesn't show that men are the same as women. No amount of word play will change that fact.

Victor Reppert said...

Surely we have to use what science has discovered to answer the questions we have.

There are some questions that we have that are philosophical in nature, which can be affected by what we learn in science. However, understandably, those are the areas where science and philosophy are difficult to distinguish. The questions about the legitimacy of string theory, or whether there is a multiverse, or when and where to make design inferences, or if there is real mental causation, are areas where science and philosophy are very difficult to pull apart. So what scientists like to do is bracket those problems and work through areas where they can get more definite results.

Of course naturalists maintain the hope that these difficult philosophical problems will clear up once we do all the bottom-up work. But I've never had any confidence that that sort of thing is likely to work. People involved in aggressive naturalism, like Dawkins, look at the success of science and think we have great reason to think it will be universal, but science is good at postponing issues that won't go away forever, and are of considerable existential concern now.

Meanwhile, the postponed issues remain issues, There are certain things about the world and about our minds which make science even possible. Restrictions in scientific methodology which make sense when you are postponing certain fundamental issues don't make sense to me when you finally try to ask those fundamental questions.

Anonymous said...

Aquinas was a deist, eh? Claiming that some causes are secondary is not the same as saying that all causes are secondary.

And what scientist claims that "all causes are secondary"? It's enough to point out that Aquinas, and many others, were entirely comfortable with framing nature as nature, a thing that acted and could be studied in its own right. So your "no gods involved" claim goes down the tubes.

Supernatural theories of life, speciation, geology, astronomy, mental illness, consciousness, morality, reason etc etc would beg to differ.

And what made them supernatural, as opposed to merely wrong? The phlogiston doesn't become a "supernatural" theory just by being wrong. The inability to define naturalism makes it just as difficult to define supernatural, and just as what used to be considered supernatural gets redefined as natural quietly, so too do past natural theories get caricatured as supernatural.

I have listed multiple criteria aside from 'no gods' that characterize a more scientific approach. You haven't addressed them.

I have pointed out that none of those criteria are somehow available only to the "naturalist" rather than the "supernaturalist", whatever that is. Oh, observation is important? Good golly, someone tell Aristotle that observation is allowed to enter into reasoning! He'll be stunned!

The existence of hermaphrodites doesn't show that men are the same as women. No amount of word play will change that fact.

You'd better hope word play is more capable than you're suggesting, because it's all naturalism has going for it anymore.

Blue Devil Knight said...

Anon: Fine. Then no men are bald. No mountains exist. There is no such thing as a planet. All these fuzzy categories imply that the clear-cut cases are also illusory. Physics is no different from biology; history is no different from political science. How helpful.

Again, pick up a science journal versus a philosophy journal. The differences are stark and obvious. Philosophers can sometimes gloss over those differences by bringing up bordeline cases (the methodological hermaphrodites), but don't let that turn you into a fool.

Gimli 4 the West said...

Why does Loftus assume the only alternative to a metal detector is Clairvoyance? If you don’t have a cellulose and ink detector available to scouring the beach with, what about the process of trying to remember where you were when the money was lost? How about interviewing beach goers and then making judgments about the truth of their stories based on intuition and experience with liars and honest men? There is a full range of methods of discovery between detectors and clairvoyance.

Anonymous said...

Vic said: The only way scientific evidence could possibly undermine religious belief would be if scientific evidence could have gone the other way and confirmed religious belief.

Without rewriting what I wrote to deal with this specific question it could have. It just doesn't.

Anonymous said...

Again, pick up a science journal versus a philosophy journal. The differences are stark and obvious. Philosophers can sometimes gloss over those differences by bringing up bordeline cases (the methodological hermaphrodites), but don't let that turn you into a fool.

I'm not denying a difference between science and philosophy. I'm saying that there's hardly anything specifically 'naturalistic' about 'methodological naturalism'. And really, if you honestly had that low an opinion of philosophy, you wouldn't bother calling yourself a naturalist. You'd eschew the labels. You wouldn't even try to group that loose criteria you have in mind as "scientific" under a philosophical heading. You'd simply say what works and why, and ignore the subject of natural or supernatural.

I'm not the one who redefined, repeatedly, just what was and wasn't "natural". I'm just pointing out its the case. Again, I'm agreeing with Victor and going one step further, pointing out just how empty "naturalism" is. It's not like I'm the one who made a mockery of the word after it needed to be "reworked" so many times in the history of science.

Blue Devil Knight said...

OK anon, we disagree about whether it is really empty. I think 'no gods' is significant. You are right, overall, that the term 'natural' is wiggly. I don't think it is quite as bad as you suggest, but I agree it is very hard to characterize precisely in a non-circular way.

I enjoy philosophy as a hobby, of course, otherwise I wouldn't be able to stand this site.

Tony Hoffman said...

"Old school chemistry was awesome, they would report the taste of compounds. They still often mention the smell."

I had heard that that was what led to the discovery of LSD -- a chemist tasting his compound.

Tony Hoffman said...

"MN simply doesn't have much to it, nor does naturalism. Sad, but true."

Funny, you don't seem sad.

Tony Hoffman said...

“People involved in aggressive naturalism, like Dawkins, look at the success of science and think we have great reason to think it will be universal, but science is good at postponing issues that won't go away forever, and are of considerable existential concern now.”

I would put forth that answering “God” is the ultimate postponement of an issue.

It appears that the criticism of science isn’t that it’s bad at answering questions, but that it doesn’t provide false positives for those questions deemed unanswerable. This reads like a compliment to me.

Tony Hoffman said...

Anon, in my experience theists who denounce methodological naturalism are unable to define "supernatural" in a way that is meaningful.

Can you define supernatural in a way that is meaningful?

Tony Hoffman said...

"I'm not the one who redefined, repeatedly, just what was and wasn't "natural". I'm just pointing out its the case. Again, I'm agreeing with Victor and going one step further, pointing out just how empty "naturalism" is. It's not like I'm the one who made a mockery of the word after it needed to be "reworked" so many times in the history of science."

I believe that the history of science is one of explaining, through natural means, that which was believed to be beyond natural explanation. If everyone were a naturalist, there'd be no need for the term, but so long as some insist that there is something called the supernatural, then science will continue to redefine (definitively) what is natural as these gaps are narrowed.

And, yes, science is supposed to be constantly "reworked," is it not? Again, this does not read to me like a criticism or failing of science, but a compliment, as the reworking only moves upward the amount of knowledge we have, not the reverse.

See: http://hermiene.net/essays-trans/relativity_of_wrong.html

Morrison said...

Atheism certainly can not be classed with "science".

For one thing, if the atheist accepts the premis3 that all existence, life, mind and reason itself can be explained by mindless processes, there there is nothing, even In Principle, that the atheist will accept as proof, for the existence of God.

How so?

Because no matter what Proof you offer, it can always be related back to some aspect of the process of mindless forces acting to account for our present development.

Hence, atheism is Unfalsifiable and, since falsifiability is a component of the Scientific Method and is used to distinguies Science from Pseudoscience, athesim is itself no "science".

And therefore, no proof will ever be enough.

Even if one were to rise from the dead.

Bobcat said...

A lot of naturalists, methodological or otherwise, rule out the following possibilities:

God
Immaterial soul
Abstract objects
Irreducible agent causation
Moral realism

A lot of naturalists rule out those phenomena because they think there are perfectly good natural explanations for what those phenomena are invoked to explain (assuming, of course, that such phenomena are posited as explanations in the first place). But it's not clear to me what a natural phenomenon is, and it's not clear to me what an explanation is. For example, imagine we come up with a neuroscience that allows us to correlate every mental event with a neural event. And imagine we can, on this basis, make pretty good, but not 100% accurate, predictions about what any given individual will do in the right set of circumstances. Does that count as a naturalistic explanation of the mental? Alternatively, imagine that we never come up with a good correlation between mental and neural events, but that we come up with a very powerful discourse in social psychological terms that allows us to make pretty good, but not 100% accurate, predictions about what any given individual will do in the right set of circumstances. Would _that_ count as a naturalistic explanation of the mental?

I bring up those questions because, as I said, I'm unclear what constitutes a successful explanation of a phenomenon, and I'm also unclear about what makes an explanation naturalistic. The way I think about it is this: imagine that there are Cartesian souls and that there is irreducible agent causation. Well, there must be some way these phenomena work, and if there's some way these phenomena work, then anyone who knows that way has an account of those phenomena. Would that fact--the fact that there is an account of those phenomena--make those phenomena natural? If so, then it's hard to imagine anyone who's not a naturalist.

I'll take my answers off the air.

Blue Devil Knight said...

Great points Tony, esp the postponement issue. Withholding judgment when there isn't sufficient evidence seems an asset that sometimes is turned into a liability.

Blue Devil Knight said...

Bobcat:
I'll take my answers off the air.

LOL

Anonymous said...

Tony Hoffman asked for a meaningful definition of "supernatural". Here's one that's always worked for me:

"Natural" refers to the sum total of creation, both physical (such as matter and energy, time and space) and non-physical (such as mathematics or gravity). Anything that can be included under the broad heading of creation (i.e., the Universe)is natural.

"Supernatural" refers to all uncreated entities, actions, influences, causes, etc.

Morrison said...

Bobcat, what you have done is to establish Naturalism as UNFALSIFIABLE, and thus taken it out of the realm of science and rendered it a Pseudoscience.

Tony Hoffman said...

Bob Prokop, I like your definition of natural.

I like the fact that you ventured forth a definition of supernatural -- this is rare. I am curious what it means to be "uncreated," however. Do you mean something that is a brute fact (the existence of the universe before the big bang)? Do you mean something that exists necessarily, like the fact that a triangle has a 3 sides? Or did you have something else in mind?

Victor Reppert said...

It isn't an objection to science that it postpones problems. Of course when you talk about false positives, that strikes me as misleading because you don't know whether they're true or false. It seems to me there is a tendency to go for false negatives because science doesn't answer the question positively.

The problem is that life doesn't wait for us to answer these questions scientifically. We have issues to decide that we can't settle by any method that fits squarely within the methodology of the sciences as currently understood.

Anonymous said...

Tony,

Cards on the table: as a Catholic theist, I of course equate the "uncreated" with God. I'm not so sure about such ideas as your example of a triangle having three sides (i.e., mathematics, logic, etc.) Frankly, my head spins when I go there, so I won't. The Creed does say, "Maker of all things visible and invisible", so presumably that might just include geometry. But I'll admit it's over my head, at least!

Once one grasps the idea behind my definition, you might see why I have no patience for atheist demands for "evidence" of the supernatural. You're just not going to get any - by definition! If you can trace an evidentiary train to something, then that something belongs to the universe (nature), and is therefore not part of the supernatural.

But this definition does not exclude the possibility of an entity from outside of nature having an effect upon that nature, which is inherently untracable to its supernatural source. A good (but still wildly imperfect) analogy here might be imagining ourselves as living inside a painting. We have no way of detecting the outside world that the artist lives in, but we can still observe that artist's handiwork, should he decide to paint something new in the picture frame (i.e., a miracle).

Tony Hoffman said...

Bob, I think you are experience a contradiction:

In one sentence you write:

“Once one grasps the idea behind my definition, you might see why I have no patience for atheist demands for "evidence" of the supernatural. You're just not going to get any - by definition!”

And then you write:

“But this definition does not exclude the possibility of an entity from outside of nature having an effect upon that nature, which is inherently untracable to its supernatural source.”

You see, I equate “evidence” with “effect.” That is why I asked for a meaningful definition of the supernatural. I can posit anything (a God, an alternate universe, bizzarro world, etc.), but if has no effect on our reality then my positing that thing is meaningless.

That is the problem with supernatural definitions. In order to not be falsified, they must be meaningless. And the ones that are meaningful (God guides the world so that the righteous are rewarded, god answers prayers, etc.) are obviously wrong, in the same way that a shirtless man saying he is hiding a cat inside his shirt is wrong.

Anonymous said...

I don't see it as a contradiction, but rather as a recognition that it's a one-way street beteen the two realms. No amount of detective work from our end will ever give us a smoking gun for the supernatural. But it is entirely within the purview of the other side to make itself known in our world (by revelation) whenever it so wishes.

We can, of course, see "evidence" all around us, as Saint Paul pointed out. But we cannot in this life follow this evidence all the way down the rabbit hole to the source. Not gonna happen.

Tony Hoffman said...

Bob, no, I think it’s still a contradiction.

If the supernatural exists and it its existence is meaningful, then we should be able to, as you say, “see ‘evidence’ all around us.” But then you also say, “. No amount of detective work from our end will ever give us a smoking gun for the supernatural.”

Think of it this way. Imagine if I said, “The evidence of unicorns is all around us.” And I followed that up with, “No amount of detective work from our end will ever give us a smoking gun for the existence of unicorns.”

Or, imagine if a scientist said, “The evidence for the atomic state of matter is all around us,” and followed that up with “No amount of detective work from our end will ever give us a smoking gun for existence of atoms.”

Please notice that the evidence doesn’t have to be scientific. Again, “The fact that I am hiding a kitten inside my shirt is obvious,” but “No amount of inspection inside my shirt will reveal a cat.”

Something has to give.

Nick said...

Bob. I think if you are a good Catholic, then the answer is that the effect must exist in the cause either formally or virtually. Whatever causes something must have inherent in it somehow the power to cause it. I would say the idea of triangularity is eternally in the mind of God that knows all there is to be known.

As for natural/supernatural, I don't really buy the distinction. There is simply existence and some existence is material in nature and some is not.

Anonymous said...

Yikes, speaking of triangles, I now have a triangular conversation!

Tony, we'll just have to either respectfully disagree, or get to the bottom of our semantic differences. There's no contradiction, once you accept the implications of the very terms we're using here. "Supernatural" has to mean what it says - above (or outside of) nature. We're literally surrounded by evidence for the Uncreated Cause behind our existence, but we are nevertheless prisoners within that created nature. No matter how cleverly we follow the trail of evidence, we will still be within the picture frame (as per my analogy).

But there are no such limitations on a being in the supernatural world (the artist's studio). He can see the whole painting in a single glance, but the characters inside the picture cannot see Him. He is quite free to alter the picture, or leave it be, at His wish. He is even free to paint Himself into the picture!

Anonymous said...

And there is nothing inherently non-logical about the idea of one-way streets, or of doors by which one may enter a room, but cannot leave by.

Anonymous said...

One more thought. Your example of evidence for unicorns is not a good analogy, because if they did exist, unicorns would still be part of the natural world, and therefore (dis)provable. Same thing for the kitten inside the shirt. You're still completely within the natural world. there's no valid analogy here.

William said...

I find it easier for some non-dualists to understand the concept of supernatural causes if we re-frame the supernatural in natural terms. One example would be if the hypothetical multiverse was a real entity, but one which was permanently closed to our observation. If the multiverse is the source of our universe, however, it would have sourced the universe as an act of supernatural creation.

Of course the multiverse is not any religion's God, and physicalists who like the idea of a multiverse would say that it too is natural. I am however referring to the idea of causes that pre-exist our universe in a way it might be understood.

Tony Hoffman said...

"Tony, we'll just have to either respectfully disagree, or get to the bottom of our semantic differences. There's no contradiction, once you accept the implications of the very terms we're using here. "Supernatural" has to mean what it says - above (or outside of) nature." 

If it's outside nature, how do we perceive it? You see, that's my problem. You appear to be declaring that something (the  supernatural) exists, but that this something is undetectable.

Your one way street analogy isn't the  problem, because we can detect the effect of a one  way street without being able to have  any effect in the  opposite  direction. It doesn't matter if we can't have any effect on the supernatural, but you need to come clean on what effect the supernatural has on us. 

"We're literally surrounded by evidence for the Uncreated Cause behind our existence, but we are nevertheless prisoners within that created nature. No matter how cleverly we follow the trail of evidence, we will still be within the picture frame (as per my analogy)."

Yes, I get the one  way street  analogy. But that's not the problem. What is the evidence for the supernatural that we are surrounded by exactly? Because everything I perceive around me looks like evidence for  the natural, not  the  supernatural. Please, be specific.

"But there are no such limitations on a being in the supernatural world (the artist's studio). He can see the whole painting in a single glance, but the characters inside the picture cannot see Him. He is quite free to alter the picture, or leave it be, at His wish. He is even free to paint Himself into the picture!"

Yup, got the one way analogy. What I don't  understand is how the characters inside the  picture know there's a painter. What is the evidence inside the painting that makes the existence of the painter clear?

Btw, unicorns are  supposed  to be magical, supernatural  creatures. That's why they're so hard to find. Some unicornists believe that there is an invisible cabal of unicorns that drives human events, but controls reality to the extent that we are unaware of their existence.

Nick said...

How do you perceive triangularity or goodness or existence itself?

Anonymous said...

As for the evidence that surrounds us, I guess I'm not asking you to regard the following as such, but they are for me (in no particular order):

Existence itself (Why is there something, rather than nothing?)

Order in the Universe

What astronomers call the "fine tuning" of the Universe.

The fact that sub-atomic particles know how to behave in various circumstances, yet that information is not stored anywhere (The hardware exists, but there is no software.)

My consciousness (self-awareness).

The habitability of the Earth, and the survival of life for the past four billion years or so.

Art, beauty, and especially music.

The inconceivable complexity of life.

The existence of evil.

I could go on, but that's enough for now. Again, I make no claims that these HAVE to be evidence for anyone else, but speaking for myself, I'm convinced.

Tony Hoffman said...

How do you perceive triangularity or goodness or existence itself?

I'll answer this, but I expect an answer to my question above in return.

I think I perceive triangularity visually, or through a mental image. But I think you're asking me about a mental process that  occurs alongside perception of something real, so if you're asking me how I perceive a mental process I'm not sure that that's a coherent question.

I think goodness is a value. I don't think that one perceives values.

I perceive existence through my experiences. I think Descartes had that one right.

Nick said...

Tony. You'll have to remind me of what the question is you want answered. If it hinges on the so-called natural/supernatural distinction, it doesn't apply to me.

You do not perceive triangularity. You perceive triangles. I can show you an isosceles, an equilateral, and a scalene. They're all different, but they all possess triangularity and you abstract triangularity from those triangles.

As for goodness, you are certainly taking a consistent opinion with atheism to say that goodness is a value for a value is a subjective opinion thrown on an objective reality. However, if you believe nothing can truly be called good, then why do you do anything at all?

Finally, for existence, do you really perceive that? It seems you see things that exist, but do you see what is existence itself. By which of your senses do you detect not existing things, but existence?

Anonymous said...

And in reply to Tony from earlier about the problem of defining "supernatural", keep in mind that's a two way street. We have the term "natural" and we have the term "supernatural". It's going to be hard to allow for "natural" being such an elastic, repeatedly redefined word and not have "supernatural" be just as touchy and elastic as a result.

The problem of defining the natural as "that which isn't supernatural" should explode into view for everyone here. In fact, I get the feeling everyone here really does see the problem with even defining the "natural", and therefore meaningfully opposing it to the "supernatural".

Though I do love the idea that someone who accepts abstract objects is not a naturalist. I guess Roger Penrose and possibly Steven Hawking aren't naturalists. (Oh wait, can't have that. Let's go back to the drawing board on what "natural" is again...) Not to mention if accepting moral realism and platonism about abstract objects suffices to make one "not a naturalist", then according to the philpapers survey supernaturalism is absolutely rife in professional philosophy, despite the lesser presence of theists.

Anonymous said...

Tony, like Nick, I am a bit confused about what question you are waiting for an answer to. I thought I answered it in my last posting???

Nick said...

Bob. If he means the definition of supernatural vs. natural, I say it's a false dichotomy. There is just existence and there are degrees of existence be it material existence or immaterial existence. There are immaterial realities that depend on matter for their actualization, such as triangles. Then there are immaterial realities that can be known through matter eventually but are not dependent on matter in any way for their being actualized, like angels and God.

Tony Hoffman said...

Sorry, running out of time tonight. Bob, kudos to you for (I think) trying to answer my questions about supernatural versus natural. Seriously, most theists just avoid the question altogether.

I have comments, questions, etc. but I won't be able to get to them til tomorrow.

Nick said...

If the anonymous poster is referring to me and I cannot tell, do note that I said I do not accept the natural/supernatural distinction.

I do believe someone who is a naturalist can accept abstract concepts like triangularity, but I think they have a harder time explaining them. I also do not hold to Platonic forms because for me, a form is only present where being is present, and for many forms, that means matter as well.

I also do think that moral realism is best accounted for in a theistic worldview rather than an atheistic worldview and I believe most of the old atheists would have certainly conceded that. Note that to say you believe something is to necessarily be consistent with your position or even have valid reasons for that position. No doubt, we all believe some things that are false, but that does not mean we don't believe them. We just believe them inconsistently or for bad reasons or some combination thereof.

Anonymous said...

Nick,

I think we're skirting dangerously close to a sterile debate over semantics here. But that said, I see no utility in defining nature as everything, with no exceptions. It renders the word meaningless. The very structure of the word supernatural requires there to be at least one thing outside of nature for the word to have any utility at all. So I guess it all depends on defining the terms. I equate nature with creation. And since God (the Creator) is Himself uncreated, He is supernatural, i.e., not within creation.

Nick said...

Bob. I did not define nature as everything. I simply said there is existence and there are degrees of existence. I am not saying there is one existent thing like Spinoza did and different modes. I am saying that things differ by the kind of being they have and the supreme being is, well, supreme in its being. There is then created existence and uncreated existence. You can speak of the cosmos as created and I agree but even if the cosmos was eternal and necessary, it would not affect theism.

The natural/supernatural distinction I believe is a technique used to give an extra burden of proof to theists when really, the naturalist needs to explain existence itself and I see no reason to think that existence itself is purely material.

Tony Hoffman said...

Bob,

I would suggest that your list of supernatural things appears to be a collection of abstract concepts and unexplained phenomena. I do not know that they should be classified outside the realm of the natural world. Is there any reason why anything on your list couldn’t be classified as an abstract concept, a mental process, or simply an unexplained phenomena?

“Tony, like Nick, I am a bit confused about what question you are waiting for an answer to. I thought I answered it in my last posting?”

My question is, how do we detect that something is supernatural? Lightning used to be thought of as a supernatural event. Now we know it is electrons, a product of nature. What distinguishes a supernatural thing from a natural one?

“You do not perceive triangularity. You perceive triangles. I can show you an isosceles, an equilateral, and a scalene. They're all different, but they all possess triangularity and you abstract triangularity from those triangles.”

Okay, but then I think you’re asking me about a mental process. Mental processes are highly complex, and little understood. But the fact that something is unexplained, as history shows, is a (very) poor indicator that it cannot be explained and/or is supernatural. But, again, what is the property of a supernatural thing that could distinguish it from a natural one? What makes something “supernatural?”

“However, if you believe nothing can truly be called good, then why do you do anything at all?”

This seems like a diversionary question, but I think it’s wrong-headed as well as it seems to imply that for something to have value it must have an absolute, objective value. This doesn’t seem true to me. (Five dollars will not feed you for a lifetime, but it will buy you lunch. By your logic above, five dollars should therefore be of no value. This thinking is obviously fallacious.)

“By which of your senses do you detect not existing things, but existence?”

I don’t believe that I detect “existence.” I think I perceive things, and that sense of perception leads to my concluding existence.

“It's going to be hard to allow for "natural" being such an elastic, repeatedly redefined word and not have "supernatural" be just as touchy and elastic as a result.”

It’s going to be even harder if you continue to refuse to offer your definition of the supernatural. You can’t criticize one side of an argument for a failing yours shares.

Nick said...

Tony.

First off, I don't accept the natural/supernatural distinction and so I have no reason to answer a question where I consider the premises to be invalid. I just say there are different degrees of existence and how one exists.

Tony:Okay, but then I think you’re asking me about a mental process. Mental processes are highly complex, and little understood. But the fact that something is unexplained, as history shows, is a (very) poor indicator that it cannot be explained and/or is supernatural. But, again, what is the property of a supernatural thing that could distinguish it from a natural one? What makes something “supernatural?”

Reply: The last part is irrelevant to me. However, you have triangularity in your mind, which is a form. If that form was existing in a part of matter in your brain, then that part would be a triangle and not the intellect. The way you grasp immaterial ideas is due to an immaterial intellect.



Tony: This seems like a diversionary question, but I think it’s wrong-headed as well as it seems to imply that for something to have value it must have an absolute, objective value. This doesn’t seem true to me. (Five dollars will not feed you for a lifetime, but it will buy you lunch. By your logic above, five dollars should therefore be of no value. This thinking is obviously fallacious.)

Reply: No. That's not my thinking. Five dollars is good, but it is not as good as five hundred dollars. There are degrees of goodness. Thus, there is no response to my question.



Tony: I don’t believe that I detect “existence.” I think I perceive things, and that sense of perception leads to my concluding existence.

Reply: Okay. I perceive things and in concluding existence itself I derive God the same way triangularity and catness are derived. It's just more complex.



Tony: It’s going to be even harder if you continue to refuse to offer your definition of the supernatural. You can’t criticize one side of an argument for a failing yours shares.

Reply: This isn't my argument so I see no reason to answer it.

Anonymous said...

Tony,

One of us misunderstood the other.

My list was not one of "supernatural things", but rather a list of evidences for the supernatural (as I defined it in an earlier posting). And I caveated that list by saying they were evidences for me. I care not whether anyone else sees them as such - they work for me.

BenYachov said...

Sounds to me like Nick is rocking classic Thomistic and Aristotelian metaphysics while Tony is stuck in Post Enlightenment Philosophical Mode(thought he may not realize it).

Let me just add this to the discussion then you may carry on. According to Hume's erroneous views (which trace back to the errors of Kant & Descartes) an alleged miracle is defined as a violation of the laws of nature because Post Enlightenment philosophy postulates there are at best only two type of causes in nature(efficient causes & material causes).

Classic philosophy OTOH postulates that there are four causes in nature (Formal, efficient, material and Final). It Also postulates that whatever is, is either Pure Act, or of necessity it is composed of Potency and Act as primary and intrinsic principles.
In this metaphysical description a miracle therefore a Potency that is Actualized directly by that which is Pure Act(i.e. which we take to be God) as opposed to being actualized secondarily in the Top down causation we call the world/Universe/Creation etc.

As Nick noted this does not require a beginning of Creation(doesn't rule it out either). But creation requires that which is Pure Actuality to keep it going from and for eternity.

Hope this helps now carry on.

Tony Hoffman said...

Nick: “First off, I don't accept the natural/supernatural distinction and so I have no reason to answer a question where I consider the premises to be invalid. I just say there are different degrees of existence and how one exists.”

I would agree, but doesn’t that make you a naturalist?



Nick: “The last part is irrelevant to me. However, you have triangularity in your mind, which is a form. If that form was existing in a part of matter in your brain, then that part would be a triangle and not the intellect. The way you grasp immaterial ideas is due to an immaterial intellect.”

This sounds like philosophical speculation about mental processes. It also sounds like Platonism. The truth is we don’t know very much about how mental processes work, but it seems just as obvious to me that the concept of triangularity is created in the mind by physical triangles, not the other way around. I don’t believe that this is resolvable philosophically.

Nick: “No. That's not my thinking. Five dollars is good, but it is not as good as five hundred dollars. There are degrees of goodness. Thus, there is no response to my question.



”

Perhaps if one accepts that Platonist forms exist, but one need not accept that. As I said before, it seems more obvious to me that Platonist forms do not exist, in which case one need not have a concept of an absolute, objective goodness in order to perceive a degree of goodness.

Nick: “Okay. I perceive things and in concluding existence itself I derive God the same way triangularity and catness are derived. It's just more complex.”

Well, no. I was using the argument for existence made famous since Descartes. I think everyone else agrees that Descartes argument from there to God fails.

Nick: "This isn't my argument so I see no reason to answer it."





My last question was to Anonymous. I was just lumping in all my responses into one comment, and not being at all fastidious about who should be attributed to what. I tried cleaning that up in this comment, and I apologize if it gave the wrong impression about your questions and the others.

Nick said...

Tony: I would agree, but doesn’t that make you a naturalist?

Reply: No. I am not one because I accept the existence of beings that are not purely material in nature.




Tony: This sounds like philosophical speculation about mental processes. It also sounds like Platonism. The truth is we don’t know very much about how mental processes work, but it seems just as obvious to me that the concept of triangularity is created in the mind by physical triangles, not the other way around. I don’t believe that this is resolvable philosophically.

Reply: No. It's not Platonic. It's fully Thomistic. Plato and Aristotle both believed in forms. Plato said triangularity could exist apart from a triangle. Aristotle said triangularity only exists where it is actualized, in triangles.

Tell me this then. Do you think you have an idea of triangularity and then you look to a three-sided figure whose degrees equal 180 and say "That will be a triangle" or do you notice all these features triangles have in common and derive triangularity from the object?



Tony: Perhaps if one accepts that Platonist forms exist, but one need not accept that. As I said before, it seems more obvious to me that Platonist forms do not exist, in which case one need not have a concept of an absolute, objective goodness in order to perceive a degree of goodness.

Reply: Not a Platonist again. Also, if things are more or less good, then that means there are cases where goodness is being actualized. Since I equate goodness with being, then I say with the fourth way following the first that there must be absolute goodness and thus absolute being.

Either something is good or else it isn't and you're just throwing the idea on to it. If it is not good in reality, don't say it is. Then try to live accordingly.



Tony: Well, no. I was using the argument for existence made famous since Descartes. I think everyone else agrees that Descartes argument from there to God fails.

Reply: Good thing I wasn't using that. I don't use the ontological argument and I am not Cartesian. He caused more problems than anything else.

Nick said...

Thanks for the compliment Ben!

BenYachov said...

>Well, no. I was using the argument for existence made famous since Descartes. I think everyone else agrees that Descartes argument from there to God fails.

Dude! Thomists HATE Cartesian horse crap!

Here is the reason why we believe Descartes OA fails(which he took from Anselm). It's likely not for the reason you think.

http://edwardfeser.blogspot.com/2010/11/anselms-ontological-argument.html


Don't mind me carry on.

Tony Hoffman said...

Nick: "No. I am not [a naturalist] because I accept the existence of beings that are not purely material in nature."

Beings like...? 

Like you, I don't think the term "supernatural" is meaningful. Can you describe the (non-natural) beings above in a meaningful way.

Nick: "Tell me this then. Do you think you have an idea of triangularity and then you look to a three-sided figure whose degrees equal 180 and say "That will be a triangle" or do you notice all these features triangles have in common and derive triangularity from the object?"

I believe the latter.

"Either something is good or else it isn't and you're just throwing the idea on to it. If it is not good in reality, don't say it is. Then try to live accordingly."

I just didn't understand this -- the part about throwing the idea on to it.

Nick said...

Tony: Beings like...?

Reply: I accept the existence of beings like angels and of course, the being of God. I accept also immaterial realities like catness, humanness, triangularity, goodness, etc. I do not accept that these realities can be actualized without being in some way in something existent.

Tony: Like you, I don't think the term "supernatural" is meaningful. Can you describe the (non-natural) beings above in a meaningful way.

Reply: You're just using the same terminology. You're saying "I don't believe in the supernatural and neither do you. I believe in only the natural. Can you explain the non-natural?" It's the same dichotomy just phrased differently. I just accept different degrees of existence.



Tony: I believe the latter.

Reply: Congratulations. You've moved away from it being a human construct which was your earlier statement. Because of this, you have said you can know reality and can draw immaterial realities from material beings.



Tony: I just didn't understand this -- the part about throwing the idea on to it.

Reply: Okay. I see a red ball and you ask me "What can you tell me about this ball?" I say "It possesses the following traits. Existence. Roundness. Redness. A certain circumference and diameter. A certain texture. Etc. I say that because I believe those exist in the object.

Now you ask "Is this ball good?"

I could say "It's good to me" but that's describing it by function instead of by what it is. How it is to me does not tell you how it is.

I could say however "It is good because it possesses existence according to its mode of being." Now I could be wrong, yeah. But I'm saying the goodness is derived from the object. I don't have an idea of goodness and instead put it on the object and then declare it good.

Make sense?

Tony Hoffman said...

Nick: "I accept the existence of beings like angels and of course, the being of God. I accept also immaterial realities like catness, humanness, triangularity, goodness, etc. I do not accept that these realities can be actualized without being in some way in something existent."

Thanks for answering my question.

I don't know much philosophy (I have never found it that interesting as taught), but I do think that philosophy can find a second life in discussions like this, because it  does become relevant. You might be under the  impression that I'm playing a game, but I am sincerely interested in the subject, and greatly appreciate it  when a question is actually answered. I probably don't agree with you ultimately, but I hate it when disagreement is achieved without sincerity.

Nick: "You're just using the same terminology. You're saying "I don't believe in the supernatural and neither do you. I believe in only the natural. Can you explain the non-natural?" It's the same dichotomy just phrased differently. I just accept different degrees of existence."

Well, I accept the natural world's existence (physical things and the supervenient), but there's a point (angels and gods) where our acceptance diverges. I don't think that's just a phrasing difference. 

Nick: "Congratulations. You've moved away from it being a human construct which was your earlier statement. Because of this, you have said you can know reality and can draw immaterial realities from material beings." 

I don't know if I said anything other than what I said. Can you please explain where my position has shifted?

Me: "I just didn't understand this -- the part about throwing the idea on to it."
Nick: "Okay. I see a red ball and you ask me "What can you tell me about this ball?" I say "It possesses the following traits. Existence. Roundness. Redness. A certain circumference and diameter. A certain texture. Etc. I say that because I believe those exist in the object.

Now you ask "Is this ball good?"

I could say "It's good to me" but that's describing it by function instead of by what it is. How it is to me does not tell you how it is. 

I could say however "It is good because it possesses existence according to its mode of being." Now I could be wrong, yeah. But I'm saying the goodness is derived from the object. I don't have an idea of goodness and instead put it on the object and then declare it good.

Make sense?"

I think you are differentiating properties and values? I just don't get your point. I thought we were discussing the supernatural. I think that overall I find statements like: "It is good because it possesses existence according to its mode of being" to be the kind of statement that I think is meaningless. It seems like the kind of language that obfuscates, rather than elucidates.

Nick said...

Tony: Thanks for answering my question.

I don't know much philosophy (I have never found it that interesting as taught), but I do think that philosophy can find a second life in discussions like this, because it does become relevant. You might be under the impression that I'm playing a game, but I am sincerely interested in the subject, and greatly appreciate it when a question is actually answered. I probably don't agree with you ultimately, but I hate it when disagreement is achieved without sincerity.

Reply: I treat my opponent as they treat me and as they treat my view. Actually, I find this statement interesting because we've said nothing scientific but everything philosophical. Philosophy dictates how we do science. Science does not dictate how we do philosophy. Both are important, but I consider philosophy helping with the more important matters.



Tony: Well, I accept the natural world's existence (physical things and the supervenient), but there's a point (angels and gods) where our acceptance diverges. I don't think that's just a phrasing difference.

Reply: They are different as to the kind of existence they have, but they do have existence, although God has it in a far different way. How do I get to them? The same way I get to triangularity and catness and other forms. It's just a longer road. Aquinas does it in each of the five ways, though if you want to discuss one, I'd do it one at a time and actually start with the first.



Tony: I don't know if I said anything other than what I said. Can you please explain where my position has shifted?

Reply: I took your former position to be we have an idea of triangularity and then we find that in the objects. In reality, we find the objects and get triangularity out of them. Now we move on to goodness and existence and we make real progress.



Tony: I think you are differentiating properties and values?

Reply: Absolutely! Good for recognizing it! Properties tell you about the object in itself! Values are a subjective preference held by a person that doesn't tell you about the object itself. If I want to know if the object is good, I don't look to my subjective ideas as if my ideas can tell me about the object itself, but I look to the object.

Tony: I just don't get your point. I thought we were discussing the supernatural.

Reply: Remember, when you say "supernatural" and "natural" I say "I don't accept the dichotomy." As far as I'm concerned, we're discussing existence and how we know the world and all existent things.

Tony: I think that overall I find statements like: "It is good because it possesses existence according to its mode of being" to be the kind of statement that I think is meaningless. It seems like the kind of language that obfuscates, rather than elucidates.

Reply: It's difficult language to be sure, but far from meaningless. For instance, if my friends start talking to me about football, and I'm not a fan at all, it's a different language to me, but not meaningless.

What I've said makes perfect sense to a Thomist and perhaps the thing to do is to start looking at Thomistic thought. I would be glad to unpack the statement I've made if you do so desire.

Tony Hoffman said...

Nick: “They are different as to the kind of existence they have, but they do have existence, although God has it in a far different way. How do I get to them? The same way I get to triangularity and catness and other forms. It's just a longer road. Aquinas does it in each of the five ways, though if you want to discuss one, I'd do it one at a time and actually start with the first.”

No, I think that all of the arguments for the existence of god fail. (If I didn’t, I wouldn’t be an atheist.) I am familiar with Aquinas’s five arguments, and their criticisms.

Along a similar line, it appears that you assert that you can “get to” to the existence of angels and gods. But that’s different than my question, which is how is their putative existence meaningful? What makes the existence of angels meaningful, in a way that an argument for the existence of invisible, magical unicorns is not?

Nick: “I took your former position to be we have an idea of triangularity and then we find that in the objects.”

I was wondering where you thought I expressed my former position. I’m honestly wondering if my thinking is confused or my writing, because that doesn’t seem like something I’d write.

Nick: “In reality, we find the objects and get triangularity out of them. Now we move on to goodness and existence and we make real progress…. Properties tell you about the object in itself! Values are a subjective preference held by a person that doesn't tell you about the object itself. If I want to know if the object is good, I don't look to my subjective ideas as if my ideas can tell me about the object itself, but I look to the object.”

Hmm. This seems like equivocation. I understand good to be a value, as you first describe above. Good is in reference to the subject. But then it seems like you want to separate the goodness of an object from the subject in the second part of your paragraph, and I think this changes the sense of good in the first.

Nick said...

Tony: No, I think that all of the arguments for the existence of god fail. (If I didn’t, I wouldn’t be an atheist.) I am familiar with Aquinas’s five arguments, and their criticisms.

Reply: If you're familiar with them, then let's start with the first way. You can demonstrate to me if you understand it. I'm skeptical now seeing as you've relied on post-Cartesian thought which Aquinas's thought cannot be understood with.

Tony: Along a similar line, it appears that you assert that you can “get to” to the existence of angels and gods. But that’s different than my question, which is how is their putative existence meaningful? What makes the existence of angels meaningful, in a way that an argument for the existence of invisible, magical unicorns is not?

Reply: And that is an irrelevant question to their existence at this point. It is not "What difference would angels make to existence?" It's first "Do angels exist?" Then when we find out they do, then we can see "What difference do they make?"



Reply: I was wondering where you thought I expressed my former position. I’m honestly wondering if my thinking is confused or my writing, because that doesn’t seem like something I’d write.

Nick: Looking back, I can see how I misunderstood, which is my bad. At least we agree that triangularity is derived from triangles. Note that triangularity in itself is non-physical but it is expressed in physical means. You can't draw me a picture of triangularity. You can draw me a picture of a triangle.



Tony: Hmm. This seems like equivocation. I understand good to be a value, as you first describe above.

Reply: That is the way moderns describe goodness. If goodness is just an idea in the mind however, then why should I believe goodness exists in the external world? How can I get from an idea in my mind of goodness to goodness outside my mind?

Tony: Good is in reference to the subject. But then it seems like you want to separate the goodness of an object from the subject in the second part of your paragraph, and I think this changes the sense of good in the first.

Reply: This was Hume's mistake. For instance, if I walk down the street and see someone mugging a little old lady, I will think that is an evil act. I am not saying I feel evil in me, therefore, the act is evil. Quite the contrary. I have an idea that the act itself is evil regardless of how I feel about it. I could struggle with something Christianity calls a sin, get really good feelings if I do that something, and still know it is not good.

My position says the evil is in the action. It's not just an idea I throw onto it. I derive evil from the action like I do triangularity from triangles.

Tony Hoffman said...

Nick: “Reply: If you're familiar with them, then let's start with the first way. You can demonstrate to me if you understand it. I'm skeptical now seeing as you've relied on post-Cartesian thought which Aquinas's thought cannot be understood with.”

Hmm. I really don’t want to get into philosophical arguments for the existence of God. As I may have mentioned earlier, philosophical arguments that don’t lead to a meaningful application (like scientific inquiry) don’t interest me much. (I agree, of course, that philosophy is primary and essential, and that it precedes science. I think that goes without saying.)

But I don’t want to ignore your question, so I’ll say that I think that the First Mover argument fails for the following reasons:

- It doesn’t establish the existence of a Christian, theistic, single, or personal god. (I do think that the argument gives me pause for a deistic creator. But I think that deism leads to agnosticism and atheism.)
- It assumes that an infinite regress is impossible. I find this no more difficult to imagine than a being that exists timelessly.
- If the universe requires a first mover, how is it that God avoids a first mover. And if God does not require a first mover, why does the universe require one?
- Like all of philosophy that does not interest me, it appears impossible to prove or disprove important premises (how do we test to see if an infinite regress is truly impossible?), or to test the argument’s premises through induction. Like all philosophy that does not interest me, it boils down to one side saying, “Works for me,” and the other saying “No it doesn’t,” with no way of resolving the impasse.

Nick: “And that is an irrelevant question to their existence at this point. It is not "What difference would angels make to existence?" It's first "Do angels exist?" Then when we find out they do, then we can see "What difference do they make?" “

And arguendo, I am allowing that they exist, but asking what difference proceeds from there? If nothing meaningful follows, the question of their existence become meaningless. (I feel the same way about alternate universes, btw.) Are you aware of invisible pink unicorn theory?

Nick: “My position says the evil is in the action. It's not just an idea I throw onto it. I derive evil from the action like I do triangularity from triangles.”

Okay. But I think you’re mistaken in your view of moral realism. I’m a moral subjectivist (who believes that objective morality is possible, but not with God). I am not a moral realist in the sense that you are, meaning that I don’t believe that morality can exist without a subject, although I do believe that desires exist, and that this is considered by some to be a kind of moral realism.

Nick said...

Tony: Hmm. I really don’t want to get into philosophical arguments for the existence of God. As I may have mentioned earlier, philosophical arguments that don’t lead to a meaningful application (like scientific inquiry) don’t interest me much. (I agree, of course, that philosophy is primary and essential, and that it precedes science. I think that goes without saying.)

Reply: I will instead focus on the first way point. You list the following on why it fails.


Tony: - It doesn’t establish the existence of a Christian, theistic, single, or personal god. (I do think that the argument gives me pause for a deistic creator. But I think that deism leads to agnosticism and atheism.)

Reply: Do you think it was meant to do that? This is the mistake Dawkins makes. Aquinas is just establishing the God of philosophy and not that of Christianity. He spends the rest of the God portion doing that.

Tony: - It assumes that an infinite regress is impossible. I find this no more difficult to imagine than a being that exists timelessly.

Reply: Aquinas does think an infinite regress is possible per accidens. See question 46 of the first book. He is fine with an eternal universe. Aquinas does not mean a first mover chronologically but fundamentally.

Tony: - If the universe requires a first mover, how is it that God avoids a first mover. And if God does not require a first mover, why does the universe require one?

Reply: A mover actualized potential in that which has potential. God has no potential to his nature and needs no actualization.

Tony:- Like all of philosophy that does not interest me, it appears impossible to prove or disprove important premises (how do we test to see if an infinite regress is truly impossible?), or to test the argument’s premises through induction. Like all philosophy that does not interest me, it boils down to one side saying, “Works for me,” and the other saying “No it doesn’t,” with no way of resolving the impasse.

Reply: We argue the same way we do everything else. Honestly Tony, as a solid Thomist, you've given me several classical misunderstandings of the First Way. I see no reason honestly to believe you do understand it. I don't mean that as an insult but a point of fact.

Tony: And arguendo, I am allowing that they exist, but asking what difference proceeds from there? If nothing meaningful follows, the question of their existence become meaningless. (I feel the same way about alternate universes, btw.) Are you aware of invisible pink unicorn theory?

Reply: I am not, but Aristotle and Aquinas would both say angels feel a hierarchy on the chain of being between essence = existence to essence + existence + matter. As a Christian, I also believe they serve God.



Tony: Okay. But I think you’re mistaken in your view of moral realism. I’m a moral subjectivist (who believes that objective morality is possible, but not with God). I am not a moral realist in the sense that you are, meaning that I don’t believe that morality can exist without a subject, although I do believe that desires exist, and that this is considered by some to be a kind of moral realism.

Reply: Tell me what you believe goodness is and how that is known.

Duke of Earl said...

Tony, forgive the shortness of reply.

- It doesn’t establish the existence of a Christian, theistic, single, or personal god. (I do think that the argument gives me pause for a deistic creator. But I think that deism leads to agnosticism and atheism.)

The Kalam argument is not intended to demonstrate the existence of the Christian God, merely the reasonableness of the existence of a creator. Depending how you use it, the creator become omnipotent (to bring something out of nothing) omniscient (to know how to construct a universe) volitional (to choose to make the universe). Now the Christian God has all those properties, so He is one possible candidate for creator, but it doesn't exclude all others.

- It assumes that an infinite regress is impossible. I find this no more difficult to imagine than a being that exists timelessly.

The Thomist formulation of the cosmological argument was written with the assumption of Aristotle's infinitely old universe. This isn't a necessary assumption.

- If the universe requires a first mover, how is it that God avoids a first mover. And if God does not require a first mover, why does the universe require one?

If we accept the conclusions of modern cosmology, the universe began with the big bang, and is expanding ever outwards. There will never be a big crunch, this is a one time only ride.

In our experience, things which begin have a cause. Things which begin are defined as contingent, dependent on something else for their existence. The universe is (by the criteria) contingent.

If the universe, on which all known things (themselves contingent) depend is itself contingent then from whence came the universe?

The cosmological argument proposes a creator, a necessary entity, as the source of the universe.

People more skilled than I with philosophy could probably give you a more refined answer.

Are you aware of invisible pink unicorn theory?

Just for the record, an invisible pink unicorn is a logically incoherent proposition, being both invisible (letting light pass through freely), and coloured (reflecting light). Since such a state of affairs cannot exist at the same time it is incoherent. Angels, whatever they might be, are not defined in such a way as to make them incoherent.

Tony Hoffman said...

"Honestly Tony, as a solid Thomist, you've given me several classical misunderstandings of the First Way. I see no reason honestly to believe you do understand it. I don't mean that as an insult but a point of fact."

Okay, and recall that I have said I'm not very interested in philosophical arguments. Because they are often not meaningful. I don't see why my misunderstandings of Aquinas's argument matter. I think my time might be better used studying a language or something.

Nick: "...Aristotle and Aquinas would both say angels feel a hierarchy on the chain of being between essence = existence to essence + existence + matter. As a Christian, I also believe they serve God."

And this seems meaningless to me. What I think would help would be evidence of angels. Without that, I find speculation about them to be as interesting as speculation invisible pink unicorns.

Nick: "Tell me what you believe goodness is and how that is known." 

Yeah, one thing that's funny to me is that being a solid Thomist as you are I don't believe you understand how that question makes little sense to me. I  wonder if you have considered the degree that your thinking is confined by your  philosophy, to the same degree that it is considered  here  that mine is.

Tony Hoffman said...

Duke: "Angels, whatever they might be, are not defined in such a way as to make them incoherent."

Great. This is what I'm asking for. Please define angels for me in a way that is not incoherent and is meaningful.

As an aside, there are many of us who think that a god who is timeless and has a mind is incoherent. So I think that the invisible pink unicorn theorists must believe they're in good company.

Nick said...

Tony: Okay, and recall that I have said I'm not very interested in philosophical arguments. Because they are often not meaningful. I don't see why my misunderstandings of Aquinas's argument matter. I think my time might be better used studying a language or something.

Reply: Aquinas himself answers in Sententia de Caelo et mundo, a reference found in Feser's work. "The study of philosophy is not about knowing what individuals thought, but about the way things are."

Why study this? Because this is the question about ultimate reality. It's about God. The most important question to ask is "Is there a God?"

Besides, would it not be best to know what the argument really says before rejecting it?



Tony: And this seems meaningless to me. What I think would help would be evidence of angels. Without that, I find speculation about them to be as interesting as speculation invisible pink unicorns.

Reply: I have already given the hierarchy of being, which is what Aristotle gives. Outside of experience by others and Scripture, God is the more fundamental question. Once God is accepted, angels easily follow based on God's revelation.



Tony: Yeah, one thing that's funny to me is that being a solid Thomist as you are I don't believe you understand how that question makes little sense to me. I wonder if you have considered the degree that your thinking is confined by your philosophy, to the same degree that it is considered here that mine is.

Reply: Yes I have. I do choose a philosophy that I believe confines me to truth and an accurate way of perceiving the world. What I ask is based on a knowledge claim.

X is good.

Okay. That's a claim about the object that it is good and not just "It is good to me." It is saying something about the object itself.

How is it known that it is good?

Duke of Earl said...

Hi Tony, I don't really have a developed theology of angels, so pardon me while I muddle through.

Literally "angel" just means "messenger of God." Consequently it is not necessary for an angel to be a spirit being. That said, their descriptions in the Bible does indicate a... I hesitate to use the word "supernatural" because the ancients didn't have the same natural/supernatural dichotomy that we adopted. Extranatural being perhaps?

They are described as being able to appear to us in human form, but disappear at will. From the visions of Daniel and John of Patmos, we could draw the conclusion that angelic beings take whatever form they wish or it may be that we may lump an entire range of beings under the heading of "angel."

There is nothing innately contradictory about angels (like the coloured invisible unicorn), but yes, without actually seeing one in the flesh I'm not sure I can come up with more than the most vague descriptions. Sorry.

I suspect that if I encountered one (based on Biblical accounts) I probably wouldn't doing much more than scrabbling on the ground anyway. They say most often when encountering humans, "fear not" suggesting fear, perhaps terror, is the most common human reaction to them.

With respect the two properties "mind" and "timeless" are not mutually contradictory properties. I am not sure that it is even necessary to conflate "eternal" and "timeless." It is simply necessary to establish, as the Bible does through prophecy, that God does not experience time in the way we do, as a linear path with no reversals. He does see the end from the beginning.

Tony Hoffman said...

Duke, I asked if you could please define angels for me in a way that is not incoherent and is meaningful.

It looks like you define them in this way:

Duke: “It is not necessary that angels be spirit beings.”

I can say this of anything known or unknown, so that really doesn’t get us anywhere.

Duke: “They are described as being able to appear to us in human form, but disappear at will. From the visions of Daniel and John of Patmos, we could draw the conclusion that angelic beings take whatever form they wish or it may be that we may lump an entire range of beings under the heading of "angel."”

Something that can take the form of whatever it wishes is a meaningless description. Like a point in every direction, it is the same as no point at all. This gets us no closer to a meaningful definition.

Duke: “There is nothing innately contradictory about angels (like the coloured invisible unicorn), but yes, without actually seeing one in the flesh I'm not sure I can come up with more than the most vague descriptions. Sorry.”

Well, the way that you’ve described them above I see no reason to doubt that any of us have seen angels, or that all that we see are angels. And that is what I mean by a meaningless definition. I just don’t see what one could possibly do with such a definition other than create a situation where the existence of such a thing can be deemed not impossible. And I think one should be very skeptical of claims that can be so easily classified.

Duke: “With respect the two properties "mind" and "timeless" are not mutually contradictory properties. I am not sure that it is even necessary to conflate "eternal" and "timeless."

Please try and explain how it is that one can “think” without time. Something’s gotta give.

Duke: “It is simply necessary to establish, as the Bible does through prophecy, that God does not experience time in the way we do, as a linear path with no reversals. He does see the end from the beginning.” “

Please give an example of a Biblical prophecy that is fulfilled outside the Bible’s account of itself.

Tony Hoffman said...

This is a try at reposting...

Nick: “Aquinas himself answers in Sententia de Caelo et mundo, a reference found in Feser's work. "The study of philosophy is not about knowing what individuals thought, but about the way things are." “

I think we agree on this point. I think philosophy is dull, and mistaught, when it is basically the history of failed ideas. It is very lively when it is not.



Nick: “Why study this? Because this is the question about ultimate reality. It's about God. The most important question to ask is "Is there a God?"”

I don’t agree with this on two levels. Number one, it’s question begging; if there is no God, the most important question isn’t about him. That would be like saying the most important question is to ask, “Are there invisible pink unicorns?” And even if there is a god, I’m not at all sure that belief in him would be more important than all those other questions that we dwell on, like what it means to be alive, etc. In other words, I don’t buy into the notion that we must define our existence in relation to God, as I believe that a God who cared about his creation would not want them to be slaves. (In other words, a benevolent God would not, perforce, want us to preoccupy ourselves with his existence. That would just be twisted.)



Nick: “Besides, would it not be best to know what the argument really says before rejecting it?”

That seems like a sensible thing to say, and it would be very hard to disagree. But I ask you, how much time have you spent exploring the arguments for these religions?

http://commonsenseatheism.com/?p=285

Do you really think “study every religion’s arguments diligently” is a good heuristic for belief in gods? If so, have you studied all of the argument for the religions listed on the link above?

Nick: “I have already given the hierarchy of being, which is what Aristotle gives. Outside of experience by others and Scripture, God is the more fundamental question. Once God is accepted, angels easily follow based on God's revelation.”

This was in response to my assertion that angels are as meaningless as invisible pink unicorns. I don’t think you’ve given me any reason to reconsider my statement.

Nick: “How is it known that it is good?”

An action is good if it increases the number of satisfied desires in the world, and bad if it decreases them. That is how I would say it is known.

I wish you might be able to stay a little more on topic, though, and consider how it is that the existence of something like angels is more meaningful than the existence of invisible pink unicorns.

Nick said...

I think I double posted unintentionally. Apologies Victor if that happened.

However, I finally got to the end of the Common Sense Atheism article.

What do I see under sources?

Wikipedia.

For a post supposed to ask if we've done any research, the poster did everything entirely online. That is indeed just lazy research. Especially if it's Wikipedia, the abomination that causes misinformation.

Nick said...

Tony: That’s subservience, and I don’t think a subservient existence is the greatest way to exist. I would also say that doing the greatest good is better than thinking about the greatest good.

Reply: Of course it isn't. The greatest way to exist is as God, and only God has that privilege. Each of us meanwhile can exist in the best way according to our mode of being. The best I can be as a human being is to be living as God designed me.

I also think doing is important, and so the greatest goods you can do are knowing and loving God, your neighbor, and yourself. The greatest joy man can have however is to see God.



Tony: In a similar way, I was raised and confirmed a Lutheran, have had deeply religious (born again) family members, roommates, best friends, have studied the New Testament in college, and taken comparative religions courses in high school and college. At some point, similar to you deciding that Chrisitianity was the one true religion, I came to determine that all religious claims were of the same bunk. I see no significant difference in our heuristics.

Reply: The difference is that my rejection of other theisms is based on an understanding that if one is true, all others are false. I have also presented you with an argument and you have given no indication you're interested in studying it, which is an odd approach if one is supposed to be seeking truth.

Perhaps you could counter by giving some positive evidence for atheism for if you can demonstrate atheism, theism is false. If I can demonstrate Christianity, all non-Christian answers are false. If I demonstrate 2 + 2 = 4, I don't have to study other answers as they're false.

Nick said...

Tony: I think you fundamentally misunderstand my approach. I am suggesting that it’s entirely reasonable (and recommendable) that, given the number of fallacious claims in this world for things like gods and demons, that those putting forth arguments for these things demonstrate that the existence of these things has some meaning. What is the effect, in reality, of knowing that gods and demons exist. Because it seems to me like there is no resulting effect to be found. I am suggesting to you that your claims, from my perspective, look exactly like the claims of all the other religions that want me to hear their arguments out.

Reply: The effect on this end is that we have an explanation for all reality, for who we are, and for why we are here. Now you say there are several fallacious claims, and I agree, but it does not follow that all are fallacious. I still have the first way which you do not understand and you have not chosen to respond to that point. Do you not want to know if you are wrong?



Tony: Okay, so you don’t understand Desirism at all. Are you sincerely interested, or do you have another point that you want to make about values?

Reply: If I do not understand something, I do want to know. Feel free to show where my point is wrong.



Tony: This is the same mistake you are making that I pointed out above. You see, we have good evidence for the existence of the appendix (we can see it, touch it, locate it on ultrasound, watch it demand blood flow from the body, etc.), and so I do not require that its existence be shown to be meaningful to me. Additionally, knowledge of the appendix’s existence, and facts that follow from its existence (like that it might become infected), are meaningful to me – my brother had his burst, for instance.

Reply: Great sympathies to hear that.

However, to get to the point, the purpose one fulfills is not a question to ask unless one is shown to exist. I am simply at this point demonstrating the existence of God. What difference He makes is secondary.

Btw, if I have time later I'll read your blog post and respond as well.

Tony Hoffman said...

Nick: “The difference is that my rejection of other theisms is based on an understanding that if one is true, all others are false.”

And my understanding is that religions, in order to be successful, often instill in their believers the notion that theirs is the one true religion.

Nick: “I have also presented you with an argument and you have given no indication you're interested in studying it, which is an odd approach if one is supposed to be seeking truth.”

In the same way that you are not interested in an argument for Zeus, I am not interested in your argument for your God. You understand this heuristic perfectly well. When someone presents me with good evidence to believe in the existence of the Christian God, I will be glad to listen to that evidence.

Nick: “Perhaps you could counter by giving some positive evidence for atheism for if you can demonstrate atheism, theism is false.”

Wow. Some positive evidence that what you claim does not exist? I can offer plenty of examples of lack of evidence, failed hypotheses, etc, but I don’t believe it’s possible to offer positive evidence for a claim like atheism. But since you bring it up, what positive evidence that God does not exist would you accept? Please answer this question, or I will assume that you were being disingenuous.

Nick: “The effect on this end is that we have an explanation for all reality, for who we are, and for why we are here.”

I’ll call BS on that one. Explanations are meaningful, and do some or all of the 9 things that I provided a link to earlier in this post. Neither you, nor anyone on this comment thread yet, has offered a meaningful explanation of the claims that something like gods and angels exist.

I am running out of time – it’s a Friday. I’ll do my best to respond to the rest of your comments later.

Tony Hoffman said...

Nick: "If I do not understand something, I do want to know. Feel free to show where my point is wrong."

I couldn't find your previous objection, but I recall it boiling down to a standard (and entirely  valid) objection of Utilitarianism. But Desirism is not Utilitarianism.

Desirism asks, "Is the fulfillment of Desire X the kind of desire (in the society in which we all live) that leads to the fulfillment of other desires? In principle, this means that desires like killing people are bad, because the person being killed would have not only their desire to live not be fulfilled, but all of the desires they had that could not be fulfilled because they were then dead. So, your scenario and calculation with the 100 rapists on an island is an incorrect method for ascertaining whether or not a desire  is good is incorrect. One needs  to ask, is rape the kind of desire that leads to an increase in desires being fulfilled? Would individuals assent to live in a society where the act of rape (their raping and being raped) was considered good? That, I believe, is the correct question to ask under Desirism.

Nick: "I still have the first way which you do not understand and you have not chosen to respond to that point. Do you not want to know if you are wrong?"

Against my better judgement go ahead and tell me a) what you think the First Way demonstrates, and b) what it demonstrates that I do not understand.

Duke of Earl said...

Then I lifted up mine eyes, and saw, and, behold, there stood before the river a ram which had two horns: and the two horns were high; but one was higher than the other, and the higher came up last. I saw the ram pushing westward, and northward, and southward; so that no beasts might stand before him, neither was there any that could deliver out of his hand; but he did according to his will, and became great. And as I was considering, behold, an he goat came from the west on the face of the whole earth, and touched not the ground: and the goat had a notable horn between his eyes. And he came to the ram that had two horns, which I had seen standing before the river, and ran unto him in the fury of his power. And I saw him come close unto the ram, and he was moved with choler against him, and smote the ram, and brake his two horns: and there was no power in the ram to stand before him, but he cast him down to the ground, and stamped upon him: and there was none that could deliver the ram out of his hand. Therefore the he goat waxed very great: and when he was strong, the great horn was broken; and for it came up four notable ones toward the four winds of heaven. Daniel 8:3-8

The ram which thou sawest having two horns are the kings of Media and Persia. And the rough goat is the king of Grecia: and the great horn that is between his eyes is the first king. Now that being broken, whereas four stood up for it, four kingdoms shall stand up out of the nation, but not in his power. Daniel 8:20-23

I point you towards Alexander the Great. Josephus has a story of Alexander coming to Jerusalem and being pleased with a reading of Daniel although I'm not confident of Josephus being a completely reliable reference.

Yes, I am well aware that because of the accuracy of its prophecies scholars have tried to date Daniel to post 300BC. However its mention of Balshazzar, a king unknown even to Herodotus, now known to be the son of Nabonidus, is an argument against dating it that late.

Also, excluding the Bible as a witness to fulfilled prophecies treats the Bible as one book, when it is a collection of books. If a later author writes of an event that fulfilled the prediction of an earlier prophet it is not a circular argument.

Also, you missed my very first definition of an angel, a messenger of God. If you encountered a messenger from God then you encountered an angel. By a variety of forms, it should be obvious that I was talking about a form capable of conveying a message.

As for the pagan deities. Have you noticed that Christians don't argue against their existence because their status, whether non-existent or merely non-authoritative, is irrelevant if Christianity is true. The first commandment forbids us placing them beside YHWH in authority whether they exist or not.

That is why, if atheists are to be taken seriously, they actually have to come up with some argument that isn't, "well what about those gods?" The Christian response is simply, "what about them?"

Remember, if you could produce an argument that showed that Zeus existed, you wouldn't be refuting Christianity, you'd be refuting atheism.

Tony Hoffman said...

Duke, I asked you for a reference being fulfilled outside the Bible, and you gave me a reference to... Daniel. As you acknowledge, this is probably the book of the Old Testament with the most historical problems, and I believe it is the leading candidate in the Old Testament for apocrypha. Even if it wasn't, using your source as evidence for it's claim is circular, and I won't accept it for the same reason you wouldn't accept my claim that I can see into the future because I have a piece of paper where I wrote that I can see into the future.

Prophecy is often claimed  by religions that seek to assume the mantle of authentic access to a power many people hope exists. It is commonly acknowledged that the Bible contains no information that could not have been known to its authors, and none that was discovered  subsequently. This isn't just my interpretation, but a matter of unbiased analysis. You've given me no reason yet to think otherwise.

Duke: "I point you towards Alexander the Great. Josephus has a story of Alexander coming to Jerusalem and being pleased with a reading of Daniel although I'm not confident of Josephus being a completely reliable reference."

Really? That's  your reference for a fulfilled prophecy? A  link to a Wikipedia entry for Alexander the Great, and an d mention that Alexander the Great was "pleased" with a reading of Daniel? How does that qualify as a prophecy?

And do you really want to put the problems with the Book of Daniel out there? From the Wikipedia entry, it appears that the best that can be said about the BoD is that it was not written purposely to deceive.

Duke: "Also, you missed my very first definition of an angel, a messenger of God. If you encountered a messenger from God then you encountered an angel. By a variety of forms, it should be obvious that I was talking about a form capable of conveying a message."

I ignored that definition because I thought it was obviously unhelpful. I am asking for a definition of angels so as to understand how  it is that they exist. Your definition of an angel as "a messenger of God" is about as helpful as my saying  that we can understand how it  is that Apollo exists by defining Apollo as "Artemis' sister." This  is just circular.

Duke: "As for the pagan deities. Have you noticed that Christians don't argue against their existence because their status, whether non-existent or merely non-authoritative, is irrelevant if Christianity is true. The first commandment forbids us placing them beside YHWH in authority whether they exist or not."

I would imagine that you also don't argue for their existence because there is no evidence that they exist. That's why I don't  argue for their existence.

Tony Hoffman said...

"That is why, if atheists are to be taken seriously, they actually have to come up with some argument that isn't, "well what about those gods?" The Christian response is simply, "what about them?" "

Okay. What about them is, Muslims say that  their God is  the one true God. Mormons say that their God is the one true God. Zororastrians (yes, I know one) claim that their God is the one true God. What about them is this: what method can you propose that will sort out your claims? Until you come up with something that works (like science works), then you have done nothing to justify what it is you believe. Seriously, it's a problem. Every believer can (because of our own biases) "prove" their religion's claims to themselves. What about them? is the question of how it is you go about proving your claims to non-believers like me.

"Remember, if you could produce an argument that showed that Zeus existed, you wouldn't be refuting Christianity, you'd be refuting atheism."

Yes. And please understand that I am not singling out Christianity as the one religions whose evidence is best accounted for through natural explanations. On the contrary.

Duke of Earl said...

You asked for an example of prophecy whose confirmation is not written in the Bible, I gave you one from Daniel and linked to an essay, which being non-controversial is as accurate as can be expected. I acknowledged that the book has criticisms, but also gave one of the reasons why the criticisms may be unjustified.

Being a "controversial" topic is the reason I wouldn't use the wikipedia entry on Daniel as authoritative. Bias isn't exclusive to religious people you know.

If you wrote on a piece of paper specific details about a king who would arise within 300 years time, giving details of his conquests, his death, and the division of his kingdom after his death, then you would have an example of the kind of prophecy that I demonstrated.

In regards to your latest objection.

Obviously a god who is the universe cannot logically be the creator of that universe. That leaves out all pantheistic religions like Hinduism, as well as godless religions like Buddhism and atheism. The classical religions like those of the Greeks and Romans must go too. In their mythology the gods were born from Khronus and he in turn came into being at the moment of creation, hence he could not be the first cause.

We're now down to a relatively short list of theistic religions with transcendent deities. Zoroastrianism, Judaism, Christianity, and Islam. I will treat Judaism and Christianity as two parts of the same tradition, although I'm sure most Jews would disagree with me.

Specific historical interventions are lacking in what I see of Zoroastrianism, that is Ahura Mazda doesn't seem to do very much. Whilst Zoroaster seems to have had many valuable insights, he doesn't appear to have the same miracle working power as Moses and Jesus.

In Judaism, whilst God doesn't act capriciously, the Biblical writers testify to specific actions and predictions attributed to YHWH. Where we are able to test them (and that is a limitation in any historical work) they seem to hold up well.

Christianity is rooted in the historical evidence of the life, death and resurrection of Jesus of Nazareth, and the theological interpretations placed on that evidence by Jesus and his apostles. Again within the limitations of historical inquiry the evidence seems to hold up well.

Islam is rooted in the revelation of Allah to Mohammed. However he makes specific claims about Jesus that are contrary to those recorded in the Bible. Since the historical evidence about Jesus goes back to the late first century and early second century, whilst Mohammed was writing in the seventh, this leaves me somewhat sceptical of his claims.

Since Mohammed did not claim to be writing his own recollections, but rather what Allah was dictating, this would lead me to regard his claims of authority as somewhat dubious.


Joseph Smith would be more convincing if there was some historical evidence to support his claims about life in the New World. Say what you like about the Bible records of Jesus, but at least there's no historical anachronisms like claiming South American tribes used steel swords prior to the arrival of the white man.

Duke of Earl said...

Since you think that imagination and mind reading are arguments worth pursuing, I think I am done with you.

Nick said...

Tony: Desirism asks, "Is the fulfillment of Desire X the kind of desire (in the society in which we all live) that leads to the fulfillment of other desires? In principle, this means that desires like killing people are bad, because the person being killed would have not only their desire to live not be fulfilled, but all of the desires they had that could not be fulfilled because they were then dead. So, your scenario and calculation with the 100 rapists on an island is an incorrect method for ascertaining whether or not a desire is good is incorrect. One needs to ask, is rape the kind of desire that leads to an increase in desires being fulfilled? Would individuals assent to live in a society where the act of rape (their raping and being raped) was considered good? That, I believe, is the correct question to ask under Desirism.

Reply: No. The question to ask is "Why should I view even the fulfillment of those desires as good?" That assumes a good end beyond those desires and why should I think one end good above another?

Nick: "I still have the first way which you do not understand and you have not chosen to respond to that point. Do you not want to know if you are wrong?"

Tony: Against my better judgement go ahead and tell me a) what you think the First Way demonstrates, and b) what it demonstrates that I do not understand.

Reply: I've already dealt with your objections to it. You can read back and see that. I have no need to repeat them.

The first way is all about the actualization of potency. All that we see is in someway a combination of act and potency. You and I exist in actuality, but we have potency in that things about us can change and then that change will be the actuality.

This is what Aquinas means by motion. He does not mean physical motion alone, although that would count.

Aquinas is also not arguing for a first mover in chronology but in being most fundamental. I am here because of my parents. My wife is here because of hers. Both of our parental sets could die and we would still be able to have children. That is a chain per accidens. Aquinas says that could go on infinitely.

The per se one can't. Picture a rock moving a leaf. The rock is moved by a stick, the stick by a hand, the hand by an arm. If you take out any part of the chain, the end movement is not there. They all must be active.

What moves the leaf however are instrumental causes and for instruments, there must be someone behind them. This is someone who is not actuality + potency as he would need an explanation himself. He is pure actuality and this everyone knows to be God.

Tony Hoffman said...

"Since you think that imagination and mind reading are arguments worth pursuing, I think I am done with you."

Okay. This seems like an entirely gratuitous statement to me ("imagination and mind reading?). If you think that your definition of angels is meaningful, and that your examples  of  prophesy support your assertions, I am content to let  readers decide who has pursued the argument to its present conclusion

Tony Hoffman said...

Reply: No. The question to ask is "Why should I view even the fulfillment of those desires as good?" That assumes a good end beyond those desires and why should I think one end good above another?

No. Desirism does not assert that anything has intrinsic value; on the contrary, it asserts that  desires are the only reason we have for actions. That is all. 

Nick: "I still have the first way which you do not understand and you have not chosen to respond to that point. Do you not want to know if you are wrong?"

Tony: Against my better judgement go ahead and tell me a) what you think the First Way demonstrates, and b) what it demonstrates that I do not understand.

Nick: I've already dealt with your objections to it. You can read back and see that. I have no need to repeat them.

I don't think you've understood my previous objection, then. If an endless  regression is possible, then the universe is a brute fact, in the same way that God is a brute fact. Something had to be; I choose the universe, because it's one less entity in my hypothesis.

And all of the above is relevant to the Original  Post  and my question (regarding how it  is that Gods and angels can be detected in a way that is meaningful) how...?

Nick said...

Tony. This is the only reply I see. If you responded to my stuff on the First Way, it didn't come up.

Tony: No. Desirism does not assert that anything has intrinsic value; on the contrary, it asserts that desires are the only reason we have for actions. That is all.

Reply: So you do an action for no reason then? You do not perceive the end as a good? Would you truly do an action if you did not perceive some good coming from that action? Note I'm not talking about intrinsic value of objects but goodness of ends.


Tony: I don't think you've understood my previous objection, then. If an endless regression is possible, then the universe is a brute fact, in the same way that God is a brute fact. Something had to be; I choose the universe, because it's one less entity in my hypothesis.

Reply: That gets more into the third way and it doesn't present a problem. Even if the universe is eternal, Aquinas says it still needs a creator. Why? Not to bring it into existence but to keep it in existence. The universe is an entity with potentiality and that potentiality is actualized by something other than the universe itself. In that sense, I rephrase Kalam.

All things that have potential depend on something else for their existing.
The universe has potential.
The universe depends on something else for its existing.

You can say the universe is one less entity, but if it cannot explain its own existence, as nothing with potential can, then you are not going far enough.

Tony: And all of the above is relevant to the Original Post and my question (regarding how it is that Gods and angels can be detected in a way that is meaningful) how...?

Reply: By using the same process whereby we detect triangularity and existence. It's not my fault if you don't want to go far enough.

Nick said...

Tony:It appears to me that the consensus of opinion of modern physicists is that the universe can be both a brute fact and the source of its own, to borrow Aristotle’s term, potentiality. (Conversely, even were I to accept Aristotle and Aquinas’s philosophical interpretation, the theist would still, I think, have to explain how it is that God has no potential and yet has a mind. In other words, I believe that the argument leads to the existence of a creator who cannot be the Christian God as described, nor the God that most theists would like to argue exists.)

Reply: What is the contradiction? God eternally knows all things and his mind never changes. He is actively thinking about all things at once.



Tony: Again, it appears that modern physics observes phenomena which do not agree with Aristotle. I find this counter-intuitive as well, but there are many scientific findings that I find counter-intuitive, and I have found that my intuitions are a poor substitute for the findings of science.

Reply: It does nothing of the sort.

http://edwardfeser.blogspot.com/2009/09/heisenberg-on-act-and-potency.html


Tony: Actually, I think you have shown yourself unwilling to go one step forward in this discussion. So far, you’ve offered some philosophical arguments for the existence of God, but that’s never been what I’ve asked for; it should be perfectly obvious that, arguendo, I am accepting the existence of God (and angels, and whatever else you think exists). My question is, assuming that these things exist, how do we detect them? Assuming that they exist, how is their existence meaningful?

Reply: Again, we detect them the same way we do triangularity, the essence of a cat, and existence itself.

Tony: The post here began with a criticism that one should not look for a lost dollar bill with a metal detector. That’s true enough, but it then introduces the question, how should one then look for a lost dollar bill? Your answer, so far, has been: argue that the dollar bill exists. As I keep trying to point out, that’s a non-sequitur, and it provides us with no way to start looking for our dollar bill. (And who knows where it could have gotten to by now.)

Reply:No. You've never asked me the question. You asked me about God's existence. Not that of a dollar bill. Perhaps we should do an old-fashioned way called digging. Maybe for God, you should do some philosophical digging.

Tony Hoffman said...

Then please explain how you could have missed these questions from me on the comments above, most of them asked directly to you:

Me: “Can you define supernatural in a way that is meaningful?”

Me: “If [supernaturalism is] outside nature, how do we perceive it?

Me: “What is the evidence for the supernatural that we are surrounded by exactly? Because everything I perceive around me looks like evidence for the natural, not the supernatural. Please, be specific.”

Me: “What is the evidence inside the painting that makes the existence of the painter clear?”

Me: “My question is, how do we detect that something is supernatural? Lightning used to be thought of as a supernatural event. Now we know it is electrons, a product of nature. What distinguishes a supernatural thing from a natural one?”

Me: “But, again, what is the property of a supernatural thing that could distinguish it from a natural one? What makes something “supernatural?”

“Nick: "No. I am not [a naturalist] because I accept the existence of beings that are not purely material in nature."
Me: “Beings like...?”

Me: “Like you, I don't think the term "supernatural" is meaningful. Can you describe the (non-natural) beings above in a meaningful way.”

Me: “I think you are differentiating properties and values? I just don't get your point. I thought we were discussing the supernatural.”

Me: “Along a similar line, it appears that you assert that you can “get to” to the existence of angels and gods. But that’s different than my question, which is how is their putative existence meaningful?”

Me: “And arguendo, I am allowing that [angels] exist, but asking what difference proceeds from there? If nothing meaningful follows, the question of their existence become meaningless.”

Me; “This is what I'm asking for. Please define angels for me in a way that is not incoherent and is meaningful.”

Me: “Duke, I asked if you could please define angels for me in a way that is not incoherent and is meaningful.”

Me: “I wish you might be able to stay a little more on topic, though, and consider how it is that the existence of something like angels is more meaningful than the existence of invisible pink unicorns.”

Me: “In the same way that you are not interested in an argument for Zeus, I am not interested in your argument for your God.”
Me: “And all of the above is relevant to the Original Post and my question (regarding how it is that Gods and angels can be detected in a way that is meaningful) how...?”

Tony Hoffman said...

Nick: “So you do an action for no reason then?”

No. I just said this – desires are the only reasons we have for actions. Why is a desire not a reason?

Nick: “You do not perceive the end as a good?”

Under Desirism, an action is good if it increases the amount of fulfilled desires.

Nick: “Would you truly do an action if you did not perceive some good coming from that action?”

A moral person, under desirism, would act in ways that tend to increase the amount of fulfilled desires, and curb those actions that tend to thwart the fulfillment of other desires.

Nick said...

Tony. I never missed any question on supernaturalism. I already answered. I don't accept the natural/supernatural distinction. I will not argue from a position I do not hold.

As for desirism, it seems you have just repeated what you said earlier. I suppose this is the point where one just repeats the argument rather than defending it hoping that makes a difference.

I still have not seen any problem I have not answered with the First Way.

Tony Hoffman said...

No, Nick, you’re resorting to lying at this point.

The OP here stated that asking science to look for the supernatural is like using a metal detector to look for a $100 bill you lost at the beach.

I then repeated the question I have been asking throughout this thread, which is, “how should one then look for a lost dollar bill?”

You responded, “You’ve never asked that question.”

I then provided a list of me asking the same question 16 times, most of them to you directly.

Your response to your obvious lie is, “I never missed any question on supernaturalism.”

I don’t mind you’re being mistaken; I look forward to discussing things with people whose beliefs are both different from mine and sincerely held. But you have shown yourself to be a dishonest interlocutor, and I won’t waste any more time with you.

I’ll leave you to the (non)responses you have made to my questions. They are a powerful testament to the character of your beliefs.